PT60.S1.Q7 - Psychologist: Research has shown that a weakened

hihihi9993hihihi9993 Member
edited October 2018 in Logical Reasoning 342 karma

Can someone please help me comprehend why "reducing stress can help weakened immune system" is same as "stress can weaken the immune system" ? It seems like invalid negation to me...

Thanks a lot!

Admin note: edited title
https://7sage.com/lsat_explanations/lsat-60-section-1-question-07/

Comments

  • redshiftredshift Alum Member
    edited October 2018 261 karma

    The passage doesn't state that reducing stress helps weakened immune systems. The passage states that (1) support groups are good because they reduce patient stress and (2) a weakened immune system increases vulnerability to cancer. The assumption required by this argument is that stress has some impact on immune systems. I.E., the author is assuming that stress has some causal connection to immune systems such that reducing stress strengthens the immune system.

    This is a perfect type of question to use the negation test. Answer (C) says "Stress can weaken the immune system." Negate this. We get: "Stress cannot weaken the immune system." Well, if stress can't weaken the immune system, then what effect would reducing stress via therapy have on cancer patients? In other words, if stress were not the cause of weakened immune systems, reducing stress in therapy would have no impact on patients' immune systems. They'd be just as likely to contract cancer with therapy or without. So the argument is destroyed.

    I'd also like to add that it seems to me that you do not properly understand illegal negations, and might do well to go back to the curriculum. An illegal negation is a formal logic flaw. It occurs when we take a conditional: a --> b, and illegally negate it to ~a --> ~b. That's not happening here. Our argument isn't using formal logic at all.

    You could also get to the right answer to this question by going through and eliminating wrong answer choices.

    (A): Negate this. Cancer patients can't learn to function well under extreme stress. This doesn't destroy the argument, because we're talking about whether reducing stress lessens one's risk of contracting cancer, not about how well one functions with cancer.

    (B) Negate this. Disease IS a biochemical phenomenon. So what? Biochemical phenomenons could still potentially be affected by therapy. Mind over matter and all that.

    (C) Is Correct for the reasons above.

    (D) Negate this. Discussing one's condition does not eliminate the stress of being in that condition. This is tempting, but our argument could still hold because, while discussing one's condition might not eliminate the stress of being in that situation, it might still reduce it. If our argument can still hold after negating a supposed necessary assumption, the answer choice isn't the correct necessary assumption.

    This is a common trap answer choice where the answer goes too far. Necessary Assumptions are assumptions that an argument depends on to be valid. Here's an example:

    Premise: I have a dog
    Conclusion: My dog is nice

    This argument doesn't need to depend on the assumption that all dogs are nice. Just that some are. Similarly, in this question, we don't need to rely on therapy completely eliminating stress, just that it helps to reduce it.

    (E) Negate this. Stress isn't a symptom of a weakened immune system. That doesn't destroy our argument, because the argument doesn't rely on stress being a symptom of a weakened immune system, but rather on it being a potential cause of weakened immune systems.

    (D) is a tempting trap answer choice. The key thing to remember in situations like this is to analyze the modifier, in this case, the phrase "being in that condition," to see how it affects the argument. Another key thing to remember is to try and pre-phrase the answer for this question beforehand. If you do, you'll be more likely to pick (C) rather than (D).

    Hope that helped!

  • hihihi9993hihihi9993 Member
    edited October 2018 342 karma

    Thank you for your comment!

    When I first saw the stimulus, I felt like I had to link 'weakened immune system' and 'reducing participant's stress level'. So, I prephrased NA answer to be something like 'stress level influences immune system' or 'reducing participants' stress level helps weakened immune system,' and tried to find AC that matches my prephrase. I was down to C and E, but realized E couldn't be right for stress is the influencing variable.

    The problem arose when I tried to break down the stimulus into parts.
    P1: Weakened immune system --> INC vulnerability to cancer
    P2: Cancer-patient support group --> DEC stress level


    C: Cancer-patient support group has therapeutic value


    NA: Cancer-patient support group --> DEC stress level --> ? --> Therapeutic value (~ DEC vulnerability to cancer)

    The given correct NA answer is "stress can weaken the immune system." For that to be established, it seems like the author is assuming [Cancer-patient support group --> DEC stress level --> Strengthened immune system --> DEC vulnerability to cancer].

    My question lies here. How are we supposed to know what leads to DEC vulnerability to cancer? Is it okay to alter [weakened immune system --> INC vulnerability to cancer] into [strengthened immune system --> DEC vulnerability to cancer]? If so, how does 'weakened immune system' suddenly switches to 'strengthened immune system'? Aren't they two different elements?

    I thought this stimulus is doing invalid negation because I perceived [A: weakened immune system --> B: inc vulnerability to cancer] to be transformed to [/A: strengthen immune system --> /B: dec vulnerability to cancer].

    Thank you again for your help! :)

  • redshiftredshift Alum Member
    edited October 2018 261 karma

    Let me answer your questions in turn.

    1. "How are we supposed to know what leads to decreased vulnerability in cancer?"

    1a. You're supposed to know this through causal reasoning. If X gives rise to situation Y, and we want to decrease Y from happening, one way to do that is to decrease the presence of X.

    1. How are we supposed to know what leads to DEC vulnerability to cancer? Is it okay to alter [weakened immune system --> INC vulnerability to cancer] into [strengthened immune system --> DEC vulnerability to cancer]?

    2a. I think I see your problem here.You are confusing sufficient with necessary conditions. Answer choice (C) is not sufficient to get us to our conclusion, and you are right to notice that. And you are also right, we cannot change "stress can weaken the immune system into [strengthened immune system -> decreases vulnerability to cancer.] Luckily, we're not trying to do that! We're not trying to find an assumption sufficient to reach our conclusion. Rather, we're trying to find an assumption that is necessary if our conclusion is valid. (C) is not sufficient, on its own and without further assumptions, to reach our conclusion. It is necessary though. There's a big difference between these two concepts, though they often overlap and though, indeed, an assumption can be both sufficient and necessary (it's not here, btw). I suggest taking a look at this link: https://plato.stanford.edu/entries/necessary-sufficient/ to clarify your understanding.

  • hihihi9993hihihi9993 Member
    342 karma

    @redshift You saved my life! Thank you so much! <3

    "If X gives rise to situation Y, and we want to decrease Y from happening, one way to do that is to decrease the presence of X."

    That inference from the causal reasoning was so difficult to digest for some reason. Thank you though. I finally understood! :)

  • redshiftredshift Alum Member
    261 karma

    No problem! Glad to help

Sign In or Register to comment.